LSAT and Law School Admissions Forum

Get expert LSAT preparation and law school admissions advice from PowerScore Test Preparation.

User avatar
 Dave Killoran
PowerScore Staff
  • PowerScore Staff
  • Posts: 5848
  • Joined: Mar 25, 2011
|
#43579
Complete Question Explanation
(The complete setup for this game can be found here: lsat/viewtopic.php?t=8478)

The correct answer choice is (C)

By looking at the templates in the setup discussion, you should be able to quickly determine which answer choice would place all seven variables without allowing any possible additional movement.

Answer choice (C) is correct because it can only exist in Template #1, where all seven variables are placed. The other four answers either allow movement within a single template, or could occur in both Template #2 and Template #3 (so they do not produce a single, fixed outcome).
User avatar
 michaelagwara
  • Posts: 1
  • Joined: Jun 17, 2021
|
#95754
If you didn't draw out the templates, would you have to test out each answer? I'm still getting into the habit of doing so.
 Rachael Wilkenfeld
PowerScore Staff
  • PowerScore Staff
  • Posts: 1358
  • Joined: Dec 15, 2011
|
#95765
Hi Michael,

Without the templates, it would be trickier. But I would focus on the answer choices that involve the conditional rule in terms of trying out answer choices. The more rules you have, the more certainty you have. So when looking to fill up the diagram, you want to use all the rules. The only rule that you would potentially not have to use in a game is a conditional rule, and because you want to use it, you need to look for answer choices that make you use that conditional rule.

That means we are looking for answer choices that have X in sales OR that have W not in P. None of the answer choices have X in sales. So now we want to look for answer choices that have W anywhere other than P. Answer choices (A) and (D) both have W in P, so that does not require anything with the conditional rule. Answer choice (C) is the only answer choice that has W in an alternative to P. That would be the first one I would try.

If W and I are both in S, that means that H, F, and Y are in P. X cannot be in sales because W would have to be in P. The only open spot for X would be in M. So our setup would have X in M, HFY in P, WIG in S. That's a complete diagram.

Hope that helps!
 mollylynch
  • Posts: 62
  • Joined: Jul 21, 2023
|
#102912
Hello, I understand why D is the correct answer but I do not understand why C is wrong. I did not use templates to create this game, however, this is what I created for C:

M: X
P: FHY
S: IWG

Why would this be wrong?
 Rachael Wilkenfeld
PowerScore Staff
  • PowerScore Staff
  • Posts: 1358
  • Joined: Dec 15, 2011
|
#103084
Hi Molly,

The correct answer for this one is answer choice (C). Is there a chance you were looking at a different question?

Get the most out of your LSAT Prep Plus subscription.

Analyze and track your performance with our Testing and Analytics Package.